Download as pdf or txt
Download as pdf or txt
You are on page 1of 52

JEE (MAIN) MOCK TEST

QUESTIONS

PHYSICS

1. The boundary surface of the media of refractive indices n1 and n2 is spherical with radius R (as shown
in the figure). Find the location of a point on axis in medium I where we can keep a point object such that
after refraction, the rays become parallel to the axis. (Use only paraxial rays.)

n2 R n2 R
(A) from the optical centre. (B) from the optical centre.
n2 n1 n1 n2
n1 R nn R
(C) from the optical centre. (D) 1 2 from the optical centre.
n2 n1 n2 n1

2. Which of the following statement is correct in connection with hydrogen spectrum?


(A) The longest wavelength in the Balmer series is longer than the longest wavelength in Lyman series
(B) The shortest wavelength in the Balmer series is shorter than the shortest wavelength in the Lyman
series
(C) The longest wavelength in both Balmer and Lyman series is equal
(D) The longest wavelength in Balmer series is shorter than the longest wavelength in the Lyman series

3. An initially stationary box of sand is to be pulled across a floor by means of a cable in which the
tension should not exceed 1100 N. The coefficient of static friction between the box and the floor is 0.35.
What should be the angle between the cable and the horizontal in order to pull the greatest possible
amount of sand?
(A) 25 (B) 19
(C) 15 (D) 23

4. Equal volumes of two immiscible liquids of densities and 2 are filled in a vessel as shown in the
figure. Two small holes are punched at depths h/2 and 3h/2 from the surface of lighter liquid. If v1 and v2
are the velocities of efflux at these two holes. Then v1 / v2 is

1 1
(A) (B)
2 2 2
1 1
(C) (D)
4 2
5. The acceleration versus time graph of a particle is shown in the following figure. The respective vt
graph of the particle is

(A) (B)

(C) (D)

6. Coils P and Q each have a large number of turns of insulated wire. When switch S is closed, the pointer
of galvanometer G is deflected towards the left (as shown in the figure). With switch S now closed, to
make the pointer of G deflect toward the right, one could

(A) Move the slide of the rheostat R quickly to the right (B) Move coil P toward coil Q
(C) Move coil Q toward coil P (D) Open S

7. In a compound microscope, the objective and the eyepiece have focal length of 1 cm and 5 cm
respectively and both are kept at a distance of 20 cm. If the final image is formed at the least distance of
distinct vision from the eyepiece, find the total magnification.
(A) 17.8 (B) 89
(C) 6 (D) 1.2

8. The figure shows a portion of a circuit. What are the magnitude and direction of the current I in the
lower right-hand wire?

(A) 4 A (B) 3 A (C) 6 A (D) 8 A

9. A particle is subjected to two simple harmonic motions, one along the x-axis and the other on a line
making an angle of 45 with the x-axis. The two motions are given by
x x0 sin t and s s0 sin t
Find the amplitude of the resultant motion.
(A) x0 2 s0 2 2 x0 s0 (B) x0 2 s0 2 2 x0 s0
(C) x0 2 s0 2 2 x0 s0 (D) x0 2 s0 2 2 x0 s0

10. Two thin rods (each of mass 0.20 kg) are joined together to form a rigid body as shown in Figure. One
of the rods has length L1 = 0.40 m, and the other has length L2 = 0.50 m. What is the rotational inertia of
this rigid body about an axis that is perpendicular to the plane of the paper and passes through the center
of the shorter rod and an axis that is perpendicular to the plane of the paper and passes through the center
of the longer rod?

(A) 0.019 kgm2 , 0.019 kgm2 (B) 0.019 kgm2 , 0.015 kgm2
(C) 0.015 kgm2 , 0.019 kgm2 (D) 0.015 kgm2 , 0.015 kgm2

11. Figure (a) given below shows a circular disc that is uniformly charged. The central z-axis is
perpendicular to the disc. Figure (b) is the graph of the magnitude of the electric field along that axis in
terms of the maximum magnitude Em at the disc surface ( zS 16.0 cm).
(a) (b)
What is the radius of the disc?
(A) 10.4 cm (B) 11.3 cm
(C) 13.9 cm (D) 14.1 cm

12. A particle is moving in a horizontal circle at a height h from the vertex in a conical funnel with
uniform speed. Find its speed.
(A) hg (B) hg cos
(C) hg tan (D) Cannot be determined

13. Figure shows a parallel-plate capacitor of plate area A = 10.5 cm2 and plate separation 2d = 7.12 mm.
The left half of the gap is filled with material of dielectric constant 1 = 21.0; the top of the right half is
filled with material of dielectric constant 2 = 42.0; the bottom of the right half is filled with material of
dielectric constant 3 = 58.0. The capacitance is

(A) 6.55 1011 F (B) 5.55 1011 F


(C) 3.55 1011 F (D) 4.55 1011 F

14. An electron (mass me ) and proton (mass 1836 me ) are moving with the same speed. The ratio of
their de Broglie wavelengths electron / proton will be
(A) 1 (B) 1836 (C) 1/1836 (D) 918

15. If frequency (F), velocity (V) and density (D) are considered as fundamental units, the dimensional
formula for momentum will be
(A) DVF2 (B) DV 2 F1
(C) D2 V2 F2 (D) DV 4 F3

16. Three polarizing sheets are placed in a stack with the polarizing directions of the first and third
perpendicular to each other. What angle should the polarizing direction of the middle sheet make with the
polarizing direction of the first sheet to obtain maximum transmitted intensity when unpolarized light is
incident on the stack?
(A) 0 (B) 30
(C) 45 (D) 60

17. A block of mass m is released from rest at a height R above the horizontal level as shown in the
figure given below. The acceleration due to gravity is g . The block slides along inside of a frictionless
circular hoop of radius R .

What is the magnitude of the normal force exerted on the block by the hoop when the block reaches the
bottom of the hoop?
(A) mg (B) 2mg
mg 2
(C) 3mg (D)
R

18. A satellite of mass m has to be launched in a circular orbit at an altitude 2R from the surface of earth.
The minimum energy required to launch the satellite is (Earth: mass M, radius R):
5GmM 2GmM
(A) (B)
6R 3R
GmM GmM
(C) (D)
2R 3R

19. Three blocks are placed on a smooth horizontal surface and along a straight line. Block-1 and Block-3
have mass m each, and Block-2 has mass M > m. Block-2 and Block-3 are initially stationary, while
Block-1 is initially moving towards Block-2 with speed v as shown. Assuming all collisions are head-on
M
and perfectly elastic, for what value of will Block-1 and Block-3 have the same final speed?
m

(A) 5 2 (B) 5 2
(C) 2 5 (D) 3 5

20. An ideal gas of mass m in a state A goes to another state B via three different processes as shown in
figure below. If Q1, Q2 and Q3 denote the heat absorbed by the gas along the three paths, then
(A) Q1 < Q2 < Q3 (B) Q1 < Q2 = Q3
(C) Q1 = Q2 > Q3 (D) Q1 > Q2 > Q3

21. At a certain distance from a point charge there is a fixed point A. At that point there is an electric field
of 500 V/m and potential of 3 kV. Then, the distance of A from the point charge is
(A) 6 m (B) 12 m
(C) 36 m (D) 144 m

22. Two wires, A and B, having resistivity A 3 10 m and B 6 10 m respectively, of same


5 5

cross section area, are joined together. If the resistance of the joined wire does not change with
temperature, then find the ratio of their lengths. Temperature coefficients of resistivity of wire A and B are
A 4 105 / C and B 6 106 / C . Assume that thermal coefficients of linear expansions are zero
for both wires.

3 10
(A) (B)
7 3
3 1
(C) (D)
10 2

23. Two bodies A and B having equal surface areas are maintaining at temperatures 10C and 20C. The
thermal radiation emitted in a given time by A and B are in the ratio
(A) 1:1:15 (B) 1: 2
(C) 1: 4 (D) 1:16

24. A straight conductor carrying current i = 5.0 A splits identical semicircular arcs as shown in below
figure. What is the magnetic field at the center C of the resulting circular loop?

(A) B = 1 (B) B = 2.5


(C) B = 0 (D) B = 2

25. Two capillary tubes of same diameter are put vertically one each in two liquids whose relative
densities are 0.8 and 0.6 and surface tensions are 60 and 50 dyne/cm, respectively. The ratio of heights of
liquids in the two tubes h2/h1 is
10 3 6 9
(A) (B) (C) (D)
9 10 10 10

26. The combinations of the NAND gates shown here in the figure are equivalent to
(A) an OR gate and an AND gate, respectively
(B) an AND gate and a NOT gate, respectively
(C) an AND gate and an OR gate, respectively
(D) an OR gate and a NOT gate, respectively

27. If an open organ pipe of length L1 and a closed organ pipe of length L2 have the same fundamental
frequency then L2 equals
(A) 4L1 (B) 2L1
L L
(C) 1 (D) 1
2 4

28. Two sources produce electromagnetic waves. Source B produces a wavelength that is three times the
wavelength produced by source A. Each photon from source A has energy of 2.1 1018 J. The energy of
a photon from source B is
(A) 7.0 1012 J (B) 7.0 1019 J
(C) 5.0 1012 J (D) 5.0 1019 J

29. A conducting loop carrying a current I is placed in a uniform magnetic field pointing into the plane of
the paper as shown in the below figure. The loop will have a tendency to
(A) contract (B) expand
(C) move toward positive x-axis (D) move toward negative x-axis

30. A TV tower has a height of 700 m. Calculate the population covered by T.V. broadcast if the
population density around the tower is 1000/km2. Radius of earth is 6.4 106 m.
(A) 28.16 lakh (B) 26.36 lakh
(C) 24.14 lakh (D) 30.36 lakh
CHEMISTRY

2 3
31. The sum of the oxidation states of Mn in MnO4 , MnO4 and MnO4 is
(A) 19 (B) 12 (C) 18 (D) 14

32. The presence of a group on benzene ring affects the further reactions of the compound. In attempts to
produce 1-bromo-3-ethylbenzene from benzene, which one of the following reaction sequence would
produce good yield of the product?
(A) (i) CH3CH2Cl + AlCl3, (ii) Br2/FeBr3
(B) (i) Br2/FeBr3, (ii) CH3CH2Cl + AlCl3
(C) (i) CH3COCl + AlCl3, (ii) Br2/FeBr3, (iii) Zn(Hg) + HCl
(D) (i) HNO3 + H2SO4, (ii) CH3CH2Cl + AlCl3, (iii) Sn/HCl, HO (iv) NaNO2, HCl/CuBr

33. If the atomic number of an inert gas element is Z, then an element with which of the following
electronic configurations will have the highest electronegativity (according to Pauling scale)?
(A) Z 1
(B) Z 2
(C) Z + 1
(D) Z + 2

34. The compound C is

(A) o-bromotoluene.
(B) m-bromotoluene.
(C) p-bromotoluene.
(D) 3-bromo-2,4,6-trichlorotoluene.

35. The water repellent characteristics of silicone is because


(A) Si-atom has no vacant d-orbital available for nucleophilic attack by water molecule
(B) Silicone chain is surrounded by organic side groups and looks like an alkane from outside
(C) The average chain lengths of silicones are very high
(D) None of these

36. The increasing order of reactivity of following alcohols towards HCl is:
F CH3

(I) OH (II)
H3C CH3

Ph

(III) OH (IV) OH
(A) II < I < III < IV
(B) I < II < III < IV
(C) IV < III < II < I
(D) I < III < II < IV
37. Two compounds Co (NO2)3 and KNO2 are mixed together in 1:3 proportion to form a complex which
produces four particles in solution. What is the correct formula for the complex?
(A) K2 [Co (NO2)6] (B) K2 [Co (NO2)4]3
(C) K3 [Co (NO2)6] (D) K2 [Co (NO2)3]3

38. Consider the two reactions given below:


I. RCONH2
2 NaOH / Br

(1) NaN3

II. RCOCl
(2) heat
(3) H 2 O

Which of the following statements is not correct about the reactions?


(A) Both reactions contain a rearrangement in which alkyl shift takes place.
(B) Both reactions pass through the intermediate called isocyanate
(C) Both reactions produce same gaseous products
(D) If R contains chiral carbon atom directly connected with carbonyl carbon atom, the configuration is
retained in the product.

39. The atomic radius of nickel is 1.24 . Nickel crystallises in a face centered cubic lattice. What is the
edge length of the unit cell?
(A) 3.5 (B) 2.48
(C) 2.86 (D) 1.75

40. Consider the rate laws given below:


(I) r k[A] ; (II) r k[A] ; (III) r k[A] ; (IV) r k
3 1/2

Which of the above rate laws cannot be used for an elementary step of reaction?
(A) I, II (B) II, III
(C) III, IV (D) II, IV

41. The treatment of 2-cyanopropane with phenyl magnesium bromide in the presence of ether and acid
produces
(A) 2-methyl-1-phenylpropanone
(B) 2-methyl-1-phenylpropanal
(C) 1-bromo-1-phenyl-2-methylpropane
(D) 2-methylpropanal

42. A galvanic cell employs the following half-reactions:


Al3 (aq) 3e Al(s); EAlo 3+ 1.66 V
Cu 2 (aq) 2e o
Cu(s); ECu 2+ 0.34 V

Which of the following statement is correct about it?


(A) Copper electrode would work as anode
3
(B) The overall cell reaction is 3Cu(s) 2Al (aq) 3Cu 2 (aq) 2Al(s)
(C) The standard cell potential of the cell is 2.00 V
(D) The mass of copper electrode decreases as electricity withdrawn from the cell

43. 20 mL of a solution containing 0.2 g of impure sample of H2O2 reacts with 0.316 g of KMnO4 in
presence of H2SO4 as per the following reaction.
KMnO4 H2O2 H2SO4 MnSO4 O2 H2O
Find the purity of H2O2 solution.
(A) 85%
(B) 82%
(C) 86%
(D) 83%

44. Which among the following amino acids is basic?


(A) Glutamine (B) Glutamic acid
(C) Histidine (D) Proline

45. When the value of the azimuthal quantum number is 3, the maximum and the minimum values of the
spin multiplicities are
(A) 4, 3
(B) 8, 1
(C) 1, 3
(D) 8, 2

46. Identify (C) in the following reaction:

(A)

(B)

(C)

(D)

47. Identify the addition polymer that would be produced from 2-chloro-2-butene:
(A)

(B)

(C)

(D)

48. Which one is correct among the following electron configurations for atoms in their ground states:
(A) [He] 2s12p3
(B) [Kr] 3d74s2
(C) [He] 2s22p4
(D) [Xe] 4f145d86s1

49. The Carbon atoms at which Bromination will take place are

(A) 1, 2 and 3 (B) 4 and 5 (C) 1, 3, 4 and 5 (D) 1, 3 and 4

50. Which of the following compounds does not follow the octet rule for electron distribution?
(A) H2O
(B) PH3
(C) PCl3
(D) PCl5

51. When one mole of an ideal gas is compressed to half of its initial volume and simultaneously heated
to twice its initial temperature, the change in entropy is
(A) CV ln 2 (B) Cp ln 2 (C) R ln 2 (D) (CV R) ln 2

52. Consider the equilibrium: 2NaHSO3 (s) Na 2SO3 (s) H2O(g) SO2 (g) . Which of the following
will affect this equilibrium?
(I) Adding NaHSO3 to the reaction vessel.
(II) Removing Na2SO3 from the reaction mixture.
(III) Adding H2O (g) to the reaction vessel.
(IV) Increasing the volume of the reaction vessel.
(A) I and III only (B) I, II and IV only (C) III and IV only (D) I, II, III and IV

53. Which of the following is not correct regarding C2 molecule?


(A) C2 molecule has been found to exist in vapor phase.
(B) It has total 12 electrons, out of which 8 electrons occupy bonding orbitals, while 4 electrons occupy
antibonding orbitals.
(C) The molecule is paramagnetic.
(D) C2 molecule contains double bond and both are bonds.

54. Which of the following pair of compounds represent enantiomers?

(A)

and

(B)

and

(C)

and

(D)

and

55. Out of which of the following pair of acids, the first one is more acidic?
(A) HSeO3 and HBrO3 (B) HClO3 and HClO2
(C) H3AsO4 and H3PO4 (D) H2CO3 and HNO3
56. Which carbanion among the following is the least stable?
(A) (CH3 )3 C
(B) (CH3 )2 HC
(C) (CH3 )H2C
(D) (C2 H5 )3 C

57. NaIO3 reacts with NaHSO3 according to equation: IO3 3HSO3 I 3H 3SO42 . The weight of
NaHSO3 required to react with 100 mL of solution containing 0.66 g of NaIO3 is
(A) 5.2 g
(B) 4.57 g
(C) 2.3 g
(D) 1.04 g

58. Which of the following colligative properties can help to determine the molar mass of a protein with
the greatest precision?
(A) Elevation in boiling point
(B) Depression in freezing point
(C) Osmotic pressure
(D) Relative lowering of vapor pressure

59. Which of the following ligand is tridentate type?

(A) Butane-1,2-diamine (bn) (B) Propane-1,3-diamine (tn)


(C) Diethylamine (dien) (D) Triethyltetraamine (trien)

60. The average, rms, and most probable velocities of gas molecules at STP increase in the order
(A) rms velocity Average velocity Most probable velocity
(B) Most probable velocity Average velocity rms velocity
(C) Average velocity rms velocity Most probable velocity
(D) rms velocity Most probable velocity Average velocity

MATHEMATICS

61. Sum of all four digit numbers whose all four digits are prime and non-repeated is
(A) 112233 (B) 113322
(C) 479952 (D) 119988

x2 y 2
62. Point P is on the ellipse 1 and O is the centre. N is the foot of the perpendicular from O onto
16 9
the tangent at P. Then the maximum area of the triangle PON is
42
(A) 42 (B)
25
7
(C) (D) 1
4
63. Let A, B and C be finite sets such that A B C and each one of the sets A B , B C and
C A has 200 elements. The number of elements in A B C is
(A) 250 (B) 200
(C) 150 (D) 300

64. Which of the following option is wrong? The equation of the plane passing through three non-
collinear points having position vectors a , b and c is (where s, t )
(A) r = (1 t s ) a + t b + s c (B) r = t a + (1 t s ) b + s c
(C) r = t a + s b + (1 t s) c (D) r = a + s b + t c

65. The total number of tangents that pass through the origin to the curve y3 9 xy 2 27 yx2 27 x3 1 0
is/are
(A) 0 (B) 1
(C) 2 (D) 3

66. Let A(5, 4) and B(7, 6) be two points. The locus of point P such that PA:PB = 2:3 is
(A) x2 + y2 2x + 48y 47 = 0
(B) x2 + y2 86x 140y + 601 = 0
(C) 5x2 + 5y2 86x 140y + 601 = 0
(D) 5x2 + 5y2 34x + 120y + 29 = 0

67. If the roots of equation (1 + m)x2 2(1 + 3m)x + (8m + 1) = 0 are in ratio 2:1, then m is
1
(A)
33
1
(B)
36
1
(C)
32
1
(D)
8

/2
(1 sin 2 x)cos 2 x
68. Evaluate: (1 3sin x cos x)
dx.
0
(A) 0
(B)
(C)
(D)1

69. Let A, B and C be square matrices of order 3 3. If A is invertible and ( A B)C BA1 , then
(A) C ( A B) A1B
(B) C ( A B) BA1
(C) ( A B)C A1B
(D) all the above
70. The equation of the circle passing through the point of contact of the direct common tangents of the
circles x2 + y2 = 16 and x2 + y2 12x + 32 = 0 is
(A) x2 + y2 + 6x 8 = 0
(B) x2 + y2 6x 8 = 0
(C) x2 + y2 4x 6 = 0
(D) x2 + y2 + 4x 6 = 0

71. If the ratio of sum of n terms of two arithmetic progressions is (7n + 5) : (2n + 3), the ratio of their
15th term is
10 208
(A) (B)
3 61
222 1
(C) (D)
65 93

72. Let a be the coefficient of x10 in the expansion of (1 x 2 )10 and b the term independent of x in the
expansion [ x (2 / x)]10 . Then a : b is equal to
(A) 1:12
(B) 1:22
(C) 6:1
(D) 1:32

73. Out of 5 men for 6 women a committee of 5 is to be formed. If the selection is at random, the
probability that the committee consists at least 3 women is
141
(A)
462
143
(B)
462
131
(C)
462
1
(D)
4

74 In ABC, angle B is a right angle. If the medians AD and BE are perpendicular to each then angle C is
1
(A) tan 1
2
1
(B) tan 1
3
(C) tan 1 ( 2)
2 2
(D) tan 1
3

75. The condition ( p q) p is


(A) a contradiction (B) a tautology
(C) neither a tautology nor contradiction (D) equivalent to (~ p ~ q)

76. If the mean of a set of observations x1, x2,...., x10 is 30 then the mean of
x1 6, x2 12, x3 18, , x10 60 is
(A) 36
(B) 60
(C) 63
(D) 90

77. Let 1 be a cube root of unity and


0
A
0
Then A2010 is equal to
(A) A
(B) A2
(C) A3
(D) 3A

3 1
78. The value of the expression is
sin 20 cos 20
2sin 20
(A)
cos 40
2sin 20
(B)
sin 40
(C) 2
(D) 4

2 1 cos x
79. lim is equal to
x 0 sin 2 x
1
(A)
2 2
1
(B)
4 2
1
(C)
8 2
1
(D)
12 2

80. The area enclosed between the curves y 2 4x and x2 4 y inside the square formed by the lines
x 1, y 1, x 4, y 4 is
8
(1)
3
16
(2)
3
13
(3)
3
11
(4)
3

81. If a line makes angles , , and with the diagonals of a cube, then find the value of
cos2 cos2 cos2 cos2 .
2 3
(A) (B)
3 2
3 4
(C) (D)
4 3

82. If a x b y c z and a, b, c are in GP, then x, y, z, are in


(A) AP
(B) GP
(C) HP
(D) AGP

83. Which of the following statements is true?


(A) A point x = x1 in the domain of f is said to be a stationary point if f ' (x1) 0.
(B) A point x = x2 in the domain of f is said to be a stationary point if f " (x2) = 0.
(C) If f is differentiable on the open interval (a, b) and if f has an absolute extremum on that interval,
then it must occur at a stationary point of f.
(D) If f is continuous on the open interval (a, b); and if f has an absolute extremum on that interval, then
it must occur at a stationary point of f.

84. If the three lines ax + y + 1 = 0, x + by + 1 = 0 and x + y + c = 0 are concurrent (where a, b and c are
1 1 1
distinct and different from 1), then find the value of .
1 a 1 b 1 c
(A) 1 (B) 2
(C) 3 (D) 4

85. If A and B are two events such that P (A/B) = P (A) then
(A) P (B/A) P (B) (B) P (A/B) = P (A)
(C) P (A/B) P (A) (D) P (A B) = P (A) + P (B)

86. A normal at P(at2, 2at1) meets the curve again at Q(at22 , 2at2 ) . If PQ subtends a right angle at the
vertex, then
(A) t12 2
(B) t22 2
(C) t1 2t2
(D) t2 2t1

87. If a function is represented parametrically by the equations


1 t 3 2
x 3 , y 2
t 2t t
then
2
dy dy
(A) x 1
dx dx
3
dy dy
(B) x 1
dx dx
3
dy dy
(C) x
dx dx
3
dy dy
(D) x 1 x
dx dx

88. If
3
x iy
2 cos i sin
then x y ?
2 2

(A) 4x 3
(B) 3x 4
(C) 4x 3
(D) 3x 4

89. The two sides of a triangle are given by the roots of the equation x2 3 3x 1 0. The angle between

the sides is . The perimeter of the triangle is 3 3 25 k . Then, k is
4

(A) 1 (B) 2
(C) 3 (D) 4

x3
90. Evaluate a 5 x5
dx
5
(A) log x5/2 a5 x5 C
2
3
(B) log x5/2 a5 x5 C
5
2
(C) log x5/2 a5 x5 C
5
2
(D) log x5/2 a5 x5 C
5
ANSWER KEYS

PHYSICS

1. (C). 2. (A). 3. (B). 4. (D). 5. (A). 6. (D). 7. (B).


8. (D). 9. (A). 10. (A). 11. (C). 12. (A). 13. (D). 14. (B)
15. (D) 16. (C) 17. (C) 18. (A) 19. (C) 20. (A) 21. (A)
22. (C) 23. (A) 24. (C) 25. (D) 26. (A) 27. (C) 28. (B)
29. (B) 30. (A)

CHEMISTRY

31. (C) 32. (C) 33. (A) 34. (B) 35. (B) 36. (B) 37. (C)
38. (C) 39. (A) 40. (C) 41. (A) 42. (C) 43. (A) 44. (C)
45. (D) 46. (D) 47. (A) 48. (C) 49. (D) 50. (D) 51. (D)
52. (C) 53. (C) 54. (C) 55. (B) 56. (A) 57. (D) 58. (C)
59. (C) 60. (B)

MATHEMATICS

61. (B) 62. (C) 63. (D) 64. (D) 65. (A) 66. (D) 67. (A)
68. (A) 69. (A) 70. (B) 71. (B) 72. (D) 73. (A) 74. (A)
75. (B) 76. (C) 77. (C) 78. (D) 79. (B) 80. (D) 81. (D)
82. (C) 83. (D) 84. (A) 85. (B) 86. (A) 87. (B) 88. (A)
89. (B) 90. (C)
SOLUTIONS

PHYSICS

1. For refraction at a single spherical surface, we have


n2 n1 n2 n1

v u R
Since the rays after refraction become parallel to the principle axis, v
n n n
1 2 1
u R
n1 R
u
n2 n1
From negative sign, we show that the object is to be placed on the left of the optical centre.
Hence, the correct option is (C).

2. The longest wavelength in Lyman series


1 1 1
R 2
L 1 4
4
L 1.3R
3R
Longest wavelength in Balmer region
1 1 1
R
B 4 9
36
B 7.2 R
5R

Therefore, B L .

Hence, the correct option is (A).

3.

From Newtons second law, we have


x : T cos f ma
y : T sin FN mg 0
Setting a = 0 and f = fs,max = s FN , we solve for the mass of the box-and-sand (as a function of angle):
T cos
m sin
g s
Differentiating w.r.t. t, and putting = m, we get (to find the angle m corresponding to the maximum
mass that can be pulled).
dm T sin m
cos m
dt g s
This leads to tan m s which (for s 0.35 ) yields m 19 .
Hence, the correct option is (B).

4.

Applying Bernoullis equation for orifice A,


h 1 2 1
P0 g v0 P0 v12
2 2 2
A0 v0 A1v1
From this, we get
v1 gh
Applying Bernoullis equation for orifice B,
h 1 1
P0 2 g 2 v02 P0 2 v22
2 2 2
A0 v0 A2 v2
From this, we get
v2 2 gh
So, v1 / v2 1 / 2
Hence, the correct option is (D).

5. We know that
dv
a
dt
dv adt
The path is parabolic starting from origin and after t1 changes its concavity.
Hence, the correct option is (A).

6. If switch S is opened, the emf would be induced in opposite direction and C would deflect in the
opposite direction, that is, in the right direction.
Hence, the correct option is (D).

7. Least distance of distinct vision D = 25 cm.


Given ve = D = 25cm
fe = 5cm

Applying lens formula for the eyepiece


1 1 1

ve ue f e
1 1 1

25 ue 5
25
ue = 4.17 cm (left of the eyepiece)
6
Objective and eyepiece are separated by distance L = 20 cm.
As measured from the objective
vo = L ue
= 20 4.17
= 15.83 cm
Apply lens formula for the objective
1 1 1

vo uo f o
1 1 1

15.83 uo 1
1 1
1
uo 15.83
15.83
uo
14.83
= 1.07cm (left of the objective)
Total magnification in a compound microscope when the final image is formed at D is given by
v D 15.83 25
M.P. o 1 = 1 89
uo f e 1.07 5
Hence, the correct option is (B).

8. By applying junction law at point 1, 2, 3, 4, 5, we get


Therefore, I = 8 A, rightward

Hence, the correct option is (D).

9. Net amplitude is
x x0 sin t s0 sin t
| x | x0 2 s0 2 2 x0 s0 (cos 45)

x0 2 s0 2 2 x0 s0
Hence, the correct option is (A).

10. (a) The rotational inertia about an axis that passes through the centre of the shorter rod is
1 1
I I1 I 2 m1L12 m2 L2 2 0.019kg m2
12 3
(b) We note that the center of the longer rod is at a distance of h = 0.25 m from the axis. The rotational
inertia is
1 1
I I1 I 2 m1 L12 m1h 2 m2 L2 2
12 12
which again yields I = 0.019 kg m2.
Hence, the correct option is (A).

11. For the uniformly charged disc, the electric field along the z- axis is
z
E 1 2
2 0 z R2

Putting z 0 , we get: Em
2 0
8.0
From the graph, 0.5Em 1
2 0 (8.0) 2 R 2

8.0
From these we get: 0.5 (8.0)2 R 2 (16.0)2
(8.0)2 R 2
Therefore, R (16.0)2 (8.0)2 13.9 cm
Hence, the correct option is (C)

12.
Let m be the mass of the particle and N be the normal reaction. Then resolving N, we have
N cos mg
mv 2
N sin
r
So,
v2 h
tan
rg r
h v2

r rg
v hg
Hence, the correct option is (A).

13.

Let C1 0 k1 A1 / 2 / 2d 0 k1 A1 / 4d , C2 0 k2 A2 / 2 / d 0 k2 A2 / 2d , and C3 0 k3A3 / 2d .


Note that C2 and C3 are effectively connected in series, while C1 is effectively connected in parallel with
the C2-C3 combination. Thus,
CC Ak A d k2 2 k3 2 0 A 2k2 k3
C C1 2 3 0 1 0 k1
C2 C3 4d k2 2 k3 2 4d k 2 k3
with A 1.05 103 m2 , d 3.56 103 m , k1 21.0 , k2 42.0 and k3 58.0, the capacitance is
(8.85 1012 C2 /N m2 )(1.05 103 m 2 ) 2(42.0)(58.0) 11
C 3 21.0 4.55 10 F
4(3.56 10 m) 42.0 58.0
Hence, the correct option is (D).
14. Mass of electron me ; mass of proton 1836 me .
Let, speed of electron = speed of proton = v.
The de Broglie wavelength of electron is
h
electron (1)
me v
and the de Broglie wavelength of proton is
h
proton (2)
1836me v
Dividing Eq. (1) by Eq. (2), we get
electron h / me v h 1836me v 1836

proton h / 1836me v me v h 1
Hence, the correct option is (B).

15. We know that:


Momentum( p) MLT 1
Now, the dimensions of density D, velocity V and frequency F are
D ML3 V LT1 F T1
So,
p D V F
a b c

a b c
MLT 1 ML3 LT 1 T 1
On comparing power coefficients, we get
a 1 3a b 1 5c 3
Therefore, a 1, b 4, c 3
So, p DV4 F3
Hence, the correct option is (D).

16. Let the angle formed by middle sheet be . According to Malus law,
I1 I 0 cos2 (For sheets 1 and 2)
and
I 2 I1 cos2 90 (For sheets 2 and 3)
Now, from the above two equations, we get:
I 2 I 0 cos2 sin 2
or
I I
I 2 0 (2cos sin )2 0 (sin 2 ) 2
4 4
I2 will be maximum when sin 2 will be maximum. Therefore,
sin 2 1
2 90
45
Hence, the correct option is (C).
17. Consider the free body diagram

Since there are no dissipative forces, the kinetic energy of the block at the bottom is equal to its potential
energy when it is released.
1 2 v2
mv mgR 2g (1)
2 R
At the bottom of the hoop, using Eq. (1)
mv 2 mv 2
N mg N mg mg 2mg 3mg
R R
Hence, the correct option is (C).

18. Radius of satellite in orbit r R 2R 3R

1 GMm
Energy of satellite in orbit: E f K f v f mv 2 Also ,
2 3R .
mv 2 GMm GMm GMm GMm GMm GMm
2 Ef Ei Ki U i 0
r r 3R 6R 3R 6R R
GMm GMm 5GMm
Energy required E f Ei
6R R 6R
Hence, the correct option is (A).

19. Initial state


M
m m
State 1 v
B2
B1 B3

After B1 collides with B2

State 2 v1 v2
B1 B2 B3

After B2 collides with B3


v1 v3
v4
State 3
B1 M m

Between State 1 and State 2

mv Mv2 mv1 (Conservation of momentum) (1)


v v2 v1 (e = 1) (2)

Between State 2 and State 3

Mv2 mv4 Mv3 (Conservation of momentum) (3)


v2 v4 v3 (e = 1) (4)

Solving equation (1) and equation (2):

v1
M m v 0 so the supposed direction is correct.
M m
2mv
v2
M m

From equation (3) and equation (4)

v3
M m v2
M m
2Mv2 2M 2mv 4Mmv
v4 0 so the supposed direction is correct.
M m M m M m M m 2

According to question, v1 v4
M m v 4Mmv 4Mm
M m M m 4Mm 0
2 2
i.e.,
M m M m M m
2

M 4 16 4

2
M M
4 1 0 m 2

m m
2 5
M
Rejecting 2 5 as it is less than 0: 2 5
m
Hence, the correct option is (C).

20. According to first law of thermodynamics,


Q W U (1)
where Q represents heat, W represents work done, U represents change in internal energy. In the cases
1,2,3, U is the same. Therefore,
Q1 W1 U
Q2 W2 U
Q3 W3 U
Now slope of pV curve represents the work done therefore for path 1 is slope is minimum than path 2
and for path3 slope is maximum, therefore,
W1 W2 W3
So, Q1 Q2 Q3
Hence, the correct option is (A).

21. Let the distance from point A be r,


kq kq
E 2
500 2
r r
kq 500r 2 (1)
kq kq
and V 3000
r r
kq 3000r (2)
Equating Eqs. (1) and (2), we get
500 r 2 3000 r
3000
r 6m
500
Hence, the correct option is (A).

22. Same cross-section AA AB A,0 3 105 m ; B,0 6 105 m


Resistance of the joined wire doesnt change with temperature.
Let the lengths be in the ratio 1: K lB Kl A ; A 4 10 / C ; B 6 10 / C ; 0 (1 T )
5 6

Al A B lB 3 105 1 4 105 T l A 6 105 1 6 106 T Kl A


R RA RB
AA AB AA AA
R should not have a T term
Therefore, coefficient of T should be zero.
l l 10
121010 A 361011 A K 0 12 36K K
AA AA 3
10
Therefore, the lengths are in the ratio 1: or 3:10.
3
Hence, the correct option is (C).

23. By Stefans law,


u AT 4
Hence T is in kelvin
4
u1 283
0.867 1:1.15
u2 293
Hence, the correct option is (A).

24. The straight segment of the wire produces no magnetic field at C. Also, the fields from the two semi-
circular loops cancel at C (by symmetry). Therefore, B at center C = 0.
Hence, the correct option is (C).

25. Relative density of first tube 0.8


Relative density of second tube 0.6
Height of liquids in first tube = h1
Height of liquids in second tube = h2
Surface tension of first tube 60 dyne cm
Surface tension of second tube 50 dyne cm
We know that
Surface tension
Height of liquid
Relative density
60
h1 (1)
0.8
50
h2 (2)
0.6
60
h1 0.8
So, 0.9
h2 50
0.6
Hence, the correct option is (D).

26. For first case, C1 A B A B (by De Morgans theorem)


The truth table is shown below:
A B A B AB AB
1 0 0 1 0 1
0 1 1 0 0 1
0 0 1 1 1 0
1 1 0 0 0 1

This is truth table for C1 = A + B, that is, OR gate


For the second case, C2 A B A B A B (by De Morgans theorem)
The truth table is shown below:
A B A B A B A B A B A B A B
1 0 0 1 1 0
0 1 0 1 1 0
0 0 0 1 1 0
1 1 1 0 0 1
This is truth table for C2 = AB, that is, AND gate

Hence, the correct option is (A).

27.
Since these fundamental frequencies are same, so should be corresponding wavelengths.
01 02 ;2L1 4L2 ; L2 L1 / 2
Hence, the correct option is (C).

28. The energy E of a photon with frequency f is


E hf
where h is Plancks constant. The frequency is related to the speed c and wavelength of the light
according to
c
f

Substituting this expression for f into above Equation gives
c
E hf h

Applying this result to both sources, we have
c c
EB h and EA h
B A
Dividing the two expressions gives
c
h
EB B A

EA h c B
A
Using the given value for EA and the fact that B 3A in this result shows that
A
EB EA (2.1 1018 J) A 7.0 1019 J
B 3A
Hence, the correct option is (B).

29. According to the given figure, the current is flowing clockwise this means the charge particle electron
is also moving clockwise.
At any point on the circular loop the direction of velocity of a charge (electron here) is tangent to the loop
at that point and the direction of magnetic field is into the plane of the paper. Therefore applying the
right-hand rule, the direction of the force will be perpendicular to both velocity and magnetic field which
will be outward direction.
Thus, the circular loop will expand under the influence of this magnetic force.
Hence, the correct option is (B).

30. Height of antenna h = 700 m, Radius of earth = 6.4 106 m.


Population density = 1000 km2 = 1000 106/m2 = 103/m2
The distance covered by the transmission is
d 2Rh or d 2 2Rh
Area covered = d2
Population covered = d2 Population density
= 3.14 2Rh 103
= 3.14 2 6.4 106 700 103
= 28.16 105 = 28.16 lakh
Hence, the correct option is (A).

CHEMISTRY

2 3
31. The oxidation states of Mn in MnO4 , MnO4 and MnO4 are +7, +6 and + 5 respectively. Thus, the
sum of their oxidation states is 18.
Hence, the correct option is (C).

32. The reaction sequence in option (A), direct alkylation followed by bromination is not a proper method
for adding bromine at meta-position to the ethyl group in benzene ring because alkyl group is ortho, para
directing. It will require the presence of a group on benzene ring which is meta directing. Also, direct
alkylation will give polyalkylated benzene ring, so we should avoid it for monoalkylation.
In the reaction sequence in option (B), isomers of 1-bromo-3-ethyl benzene would be produced because
bromine shows +R effect due to which it is ortho, para-directing group.
The sequence which will give good yield is given in option (C).

In the reaction sequence in option (D), NO2 group is deactivating (more than COCH3) when attached
with benzene ring, so, Friedel Craft alkylation is not possible in the presence of such a highly deactivating
group.
Hence, the correct option is (C).

33. The element having atomic number (Z 1) is a halogen and has the highest electronegativity in that
period.
Hence, the correct option is (A).

34. The reaction involved is

Hence, the correct option is (B).

35.
The Si-O-Si skeleton is protected by water repelling alkyl groups that orient themselves towards the
surface creating a water proof structure, like an alkane.
Option (A) is incorrect as Si has vacant 3d orbital.
Option (C) is incorrect as there is no relation between chain length and water repelling characteristics.
Hence, the correct option is (B).

36. Reactivity of alcohols towards HCl is decided by formation of more stable carbocation. Greater the
stability of carbocation, more will be the reactivity.
Among the given four alcohols, (IV) makes the most stable carbocation as it is benzylic carbocation
which is resonance stabilized. Alcohols (I), (II) and (III), all form secondary carbocations, of which that
formed by (III) is most stable due to positive inductive effect of two methyl groups. The carbocation
formed by (II) is more stable than that from (I) due to lower negative inductive effect of fluoro group.
From the above observations the correct order of formation of stable carbocation is the same as the
reactivity towards HCl. Thus, the order is

> > >

or I < II < III < IV


Hence, the correct option is (B).

37.

K3 Co NO2 6 3 K + + Co NO2 6
3-

Co(NO2 )3 +3KNO2

From the reaction, we can see that option (C) is correct.


For option (A): K 2 [Co(NO2 )6 ] 2K + +[Co(NO2 )6 ]2
Produces only3 ions

For option (B): The formula K2 [Co (NO2)4]3 itself is wrong.


For option (D): The formula K2[Co(NO2)3]3 itself is wrong.
Hence, the correct option is (C).

38. (I). Hofmann Rearrangement


(II). Curtius Rearrangement

The reactions can be summarized as


RCONH 2 Br2
KOH
RNCO
H2 O
RNH 2 CO2
RCOCl
(1) NaN3 , (2) Heat
(3) H 2 O
RNH 2 CO2 N 2
Both the reactions are almost the same, both involve alkyl shift without inversion of configuration; both
reactions pass through the isocyanate intermediate; and both produce gaseous products. However, in the
first reaction only CO2 is evolved, while in the second reaction two gaseous products, N2 and CO2 are
evolved.
Hence, the correct option is (C).

39. In fcc, atoms touches each other on the face diagonal of the cube. Thus,
4 1.24
4r 2 a a 3.5
2

Hence, the correct option is (A).

40. The elementary reaction is the simplest form of the reaction, and rate of these reactions is directly
proportional to the concentration of species participating in the reaction.
However, the probability of colliding more than 3 species at a time is very low, so, an order > 3 is not
possible in this case.
In elementary reactions, 1, 2, or 3 species can participate at a moment. Thus, order = 1, 2, or 3 for the
elementary reaction.
Zero and fractional orders are not possible for elementary reactions, as these orders imply a complex
reaction.
Hence, the correct option is (C).

41. A nitrile reacts with Grignard reagent followed by hydrolysis to yield a ketone.
O

CH3CH - CN + C6 H5MgBr
(1) Et 2O
CH3CHCC6 H5 Mg(Br)(OH) + NH3
(2) H3O+

CH3 CH3

Hence, the correct option is (A).

42. The reduction potential of aluminium is negative, but that of copper is positive, so Cu 2+ will be
reduced and Al will be oxidized in the cell reaction.
The Al electrode will work as anode and copper electrode will work as cathode. Since Al is oxidized at
anode, its mass will decrease on withdrawing electricity from the cell. Therefore,
o
Ecell Ecathode
o
Eanode
o
0.34 (1.66) 2.00V
Hence, the correct option is (C).

43. The complete balanced reaction is:


2KMnO4 5H2O2 3H2SO4 2MnSO4 K 2SO4 5O2 8H2O
From the balanced reaction, we have
5 mol of H2O22mol of KMnO4

0.316
Moles of KMnO4 =
158
So,
5 0.316
Moles of H2O2 required =
2 158
5 0.316
Grams of H2O2 = 34 0.17 g
2 158
0.17
Thus,% purity of H2O2 = 100 85%
0.2
Hence, the correct option is (A).

44. The structures of the amino acid given in each of the options above are as follows:
It is clear from the structures of the given amino acids that only histidine contains a basic group in the
side chain.
Hence, the correct option is (C).

45. Spin multiplicity is 2S + 1. l = 3 corresponds to s, p, d, and f orbitals. Maximum multiplicity for the f
7
orbitals (7 electrons) 2S 1 2 1 8
2
1
Minimum multiplicity for the f orbital (1 electron) 2S 1 2 1 2
2
Hence, the correct option is (D).

46. The reaction is

(A) (B) (C)


Hence, the correct option is (D).

47. The addition polymerization reaction of 2-chloro-2-butene is


Cl H
| |
CH3 C CH CH 3 C C
| | |
Cl
CH3 CH 3 n
Hence, the correct option is (A).

48.
(A) 2p cannot be filled unless 2s is completely filled.
(B) [Kr] should be replaced by [Ar]
(C) is correct.
(D) The filling of orbitals is incorrect. According to Aufbau principle, 6s should be filled first followed by
4f and 5d.
Hence, the correct option is (C).

49. N-Bromosuccinimide (NBS) is frequently used to brominate allylic positions because it allows a
radical substitution reaction to be carried out without subjecting the reactant to a relatively high
concentration of Br2 that could add to its double bond.
Carbon atoms at positions 1, 3 and 4 are allylic carbons. The carbon atom at position 5 is not allylic since
it becomes acidic due to presence of carbonyl carbon at the alpha position.
Hence, the correct option is (D).

50. P has 10 electrons in its valence shell in PCl5.

Hence, the correct option is (D).

51. The change in entropy is given by


T V 1
S CV ln 2 R ln 2 CV ln 2 R ln CV ln 2 R ln 2 (CV R)ln 2
T1 V1 2
Hence, the correct option is (D).

52. The reaction exhibits a heterogeneous equilibrium, so, solid state species would not contribute to the
equilibrium constant.
2NaHSO3 (s) Na 2SO3 (s) H2O(g) SO2 (g)
On the product side, there are SO2 and H2O gases which contribute to the equilibrium constant. Any
factor that changes the concentration of these gases will affect the equilibrium. Therefore, addition of
water vapor and increasing the volume of vessel will affect the equilibrium.
Hence, the correct option is (C).

53. C2 = ( 1s)2 ( *1s)2 ( 2s)2 ( *2s)2 ( 2 px2 2 p2y )


Four electrons are present in 2 molecular orbitals; that is why double bond contains both bonds.
Molecule is diamagnetic as it does not have any unpaired electron.
Hence, the correct option is (C).

54. For option (A): The molecules have plane of symmetry, hence, they are optically inactive and both
are identical.

For option (B): In cis and trans- decalin, the configuration around one carbon atom is changed, hence,
they are diastereomers.
In option (C): The configuration around all chiral carbons is inverted hence they are enantiomer.

and

In option (D): Molecule has a plane of symmetry, hence, it is optically inactive meso isomer and both
are identical

Hence, the correct option is (C).

55. In oxoacids, acidity depends on the electronegativity of central atom and the number of oxygen atoms
attached.
If central atom is more electronegative, then its acid strength is more and presence of more oxygen atoms
causes more acidity.
In the second option, HClO3 contains more oxygen atoms than HClO2, so HClO3 is more acidic.
(A) HSeO3 < HBrO3 (B) HClO3 > HClO2 (C) H3AsO4 < H3PO4 (D) H2CO3 < HNO3
Hence, the correct option is (B).

56. The relative order of stability of carbanion is:


Methyl > Primary > Secondary > Tertiary
This order can be explained on the basis of +I effect of the alkyl groups. Greater the number of alkyl
groups attached to the C bearing the negative charge, greater will be the electron donating effect and
hence the negative charge will be intensified. Therefore, tertiary carbanion carrying three methyl groups
is least stable.
Hence, the correct option is (A).

57. In the reaction


I5 6e I
S6 2e S4
Therefore,
M equiv NaHSO3 M equiv NaIO3
N1V1 (NaHSO3 ) N 2V2 (NaIO3 )
w 0.66
100 100
104 / 2 198 / 6
w 1.04g
Hence, the correct option is (D).

58. Osmotic pressure is used for molar mass determination of protein; it is given by
wRT

M V
where w is weight of protein, R is the gas constant, T is the temperature, M is the molecular mass of
protein and V is the volume of the solution.
Hence, the correct option is (C).

59. (A) Butane-1,2-diamine (bn):

(B) Propane-1,3-diamine (tn):

(C) Diethylamine (dien):

(D) Triethyltetraamine (trien):


Hence, the correct option is (C).

60. Molecular velocities of gases are as follows:


(A) The rms (root mean square) velocity
3RT
C
M
8RT
(B) Average velocity
M
urms = 1.085 uavg
2RT
(C) Most probable velocity
M
The ratio is
Most probable velocity: Average velocity : rms velocity
8
2: : 3 1:1.128 :1.224

Hence, the correct option is (B).

MATHEMATICS

61. Only prime digits are 2, 3, 5, 7


Each digit at any place occurs 6 times hence sum of each column is:(2 +3 + 5 + 7) 3! = 17 6 = 102
Hence, the sum is given by 102 103 + 102 102 + 102 10 + 102= 102 1111 = 113322
Hence, the correct option is (B)

1
62. See the following figure. Area of PON (ON )( NP) .
2

To find ON shown in the figure, P (a cos , b sin ) P (4cos ,3sin )


xx yy x cos y sin
The equation of tangent at point P is: 1 1 1 1
16 9 4 3

1 12
Therefore, ON
cos2 sin 2 9cos2 16sin 2

16 9
Note
To find NP, write the equation of normal at point P.
OM be perpendicular to normal at P OM NP

x x1 y y1
The equation of normal at point P(x1, y1) is 2

x1 / a y1 / b2
This implies that the equation of normal at(4 cos , 3 sin ) is

x 4cos y 3sin sin cos


( x 4cos ) ( y 3sin ) 4 x sin 3 y cos 7sin cos
4cos / 16 3sin / 9 3 4

7sin cos
Therefore, OM NP
16sin 2 9cos 2

1
Therefore, the area of ONP (ON )( NP) is
2
1 12 7sin cos 42


2 9cos 2 16sin 2 16sin 9cos 9cot 16 tan
2 2

9cot 16 tan
9cot 16 tan 9cot 16 tan 24 for 0 ( AM GM )
Now, 2 2

42 7
Hence, Area
24 4
7
Therefore, the maximum value of area .
4
Hence, the correct option is (C).

x2 y2 x x1 y y1
Quick Tip: The equation of normal at point P(x1, y1) on ellipse 2
2
1 is: 2

a b x1 / a y1 / b2

63. Let n( X ) denote the number of elements in X. Then,


n( A B C) n( A) n( B) n(C) n( A B) n( B C) n(C A) n( A B C).
n( A) n( A B) (since A B C )
Now, A B ( A B) ( B A) ( A B) ( A B). Therefore,
n( A B) n( A B) n( A B) n( A) n( B) 2n( A B)
and 600 n( A B) [n( A) n(B) 2n( A B)] 2 n( A) n( A B)
Therefore n( A B C ) n( A) n( A B)
600
300
2
Hence, the correct option is (D).

Alternate Solution: Draw the Venn diagram as follows:

The shaded part is A B C which is given to be empty. Let a, b, c denote


n( A ( B C)), n( B (C A)), n(C ( A B)) , respectively. Let x, y, z denote the number of
elements in ( A B) C , ( B C ) A , (C A) B , respectively. Then
n( A B C ) a b c x y z
We are given that
200 n( A B) (a z) (b y) 200 n(B C ) (b x) (c z ) 200 n(C A) (c y) (a x)
Adding the above three equations, we get that: 600 2(a b c) 2( x y z) 2n( A B C)
Hence, n( A B C ) 300.

64. Let us consider option (A):


r = (1 t s ) a + t b + s c = a + ( b a )t + s ( c a )= a + t ( AB) + s ( AC )
Here, r represents position vector of any point on the plane containing points A( a ), B (b) and C (c )
Similarly, options (B) and (C) are also correct.
Option (D) represents a plane passing through the points having position vectors
a , a + b (s = 1 and t = 0) and a + b c (s = 1 and t = 1)
Hence, the correct option is (D).

65. Using the method of perfect cubes, we can reduce the given equation to smaller factors
Therefore,
y3 9 xy 2 27 yx2 27 x3 1 0 ( y 3x)3 1 0 ( y 3x)3 1 y 3x 11/3 1 (as x and y are real)

which is the equation of a straight line not passing through the origin. From this, we conclude that no
tangent passing through the origin can be drawn to this curve and therefore, the answer for this problem is
0.
Hence, the correct option is (A).
Quick Tip: The given equation can be written in the form (a b)3 1 0.

Alternate Solution: Differentiating the given curve, we get


dy
3y2 9[ y 2 x(2 yy ')] 27( x 2 y ' 2 xy) [27(3)( x 2 )] 0
dx
dy dy 9 y 2 54 xy 81x 2
Therefore, (3 y 2 18xy 27 x 2 ) 81x 2 9 y 2 54 xy 3
dx dx 3 y 2 18 xy 27 x 2
This implies that the given equation is an equation of a straight line that does not pass through the origin
to the curve. Therefore, the total number of tangents to this curve passing through the origin is 0.

66. Let P be (x, y). Then


PA:PB = 2:3 3PA = 2PB
9(PA)2 = 4(PB)2
9[(x 5)2 + (y + 4)2] = 4 [(x 7)2 + (y 6)2]
5x2 + 5y2 34x + 120y+ 29 = 0
Hence the equation of the locus is 5x2 + 5y2 34x + 120y + 29 = 0. This equation represents circle.
Hence, the correct option is (D).

67. If the ratio of roots of a quadratic equation is 2:1, then the roots are 2 and .
The sum of the roots is
2(3m 1)
3
(m 1)
The product of the roots is
8m 1
2 2
m 1
Eliminating , we get
2
2(3m 1) (8m 1)
2
3(m 1) ( m 1)
8(3m 1) 2 9(8m 1)(m 1)
8(9m 2 6m 1) 9(8m 2 9m 1)
72m 2 48m 8 72m 2 81m 9
33m 1
1
m
33

Quick Tip: If the roots of a quadratic equation of form ax2 + bx + c = 0 are in the ratio m:n, then

b 2 ( m n) 2

ac mn
Hence, the correct option is (A).

68. We have
/2
(1 sin 2 x) cos 2 x
I (1 3sin x cos x)
dx
0


/2 1 sin 2 x cos 2 x
2 2

dx
1 3sin 2 x cos 2 x
0

/2
(1 sin 2 x)( cos 2 x)
1 3sin x cos x
dx
0
I
which implies that
2I 0
I 0
Hence, the correct option is (A).

69. We have
( A B)C BA1 AC BC BA1
AC BC BA1 AA1 I (Unit matrix)
( A B)C ( A B) A1 I
( A B)(C A1 ) I
Therefore, C A1 is the inverse of A B. This implies
(C A1 )( A B) I
C ( A B) I A1 A A1B
C ( A B) A1B
Hence, the correct option is (A).
70. Let S2 be the external centre of similitude. Therefore, S2 divides the line joining O (0, 0) and A(6, 0)
externally in the ratio 2:1. Hence S2 = (12, 0). Therefore, equation of the chord of contact of S2(12, 0) with
respect to x2 y 2 16 is
L 3x 4 0 (1)
Equation of the chord of contact of S2(12, 0) with respect to x2 y 2 12 x 32 0 is
L 3x 20 0 (2)
Now, equation of the circle passing through the points of intersection of x2 + y2 16 = 0 and L = 0 is of
the form
x2 y 2 16 (3x 4) 0 (3)
Equation of the circle passing through the intersection of x + y 12x + 32 = 0 and L = 0 is
2 2

x2 y 2 12 x 32 (3x 20) 0 (4)


Equations (C) and (D) represent the same circle 3 = 3 12 and 4 16 = 20
+ 32 = 4 and 5 = 12. Therefore, = 2 and = 2 so that the equation of the
required circle is x2 + y2 6x 8 = 0.

Hence, the correct option is (B).

7n 5
71. The ratio of sums of n terms of the two arithmetic progressions is :
2n 3
(n / 2)[2a1 (n 1)d1 ] 7n 5 a [(n 1) / 2]d1 7n 5
That is, 1
(n / 2)[2a2 (n 1)d 2 ] 2n 3 a2 [(n 1) / 2]d 2 2n 3
a1 14d1
Therefore, the ratio of their 15th term is :
a2 14d 2
n 1 a 14d1 7 29 5 208
Let 14 ; therefore, n 29 . Thus, 1
2 a2 14d 2 2 29 3 61
Hence, the correct option is (B).

72. We have x10 ( x 2 )5 . The coefficient of x10 is 10


C5 .
Independent term in the expansion [ x (2 / x)] 10
is 25 10C5 . Therefore
a : b 1: 25 1: 32

Hence, the correct option is (D).

73. E: Event that committee consists of at least 3 women.


The favorable cases E: Women Men No. of selections
3 2 6
C3 5 C2
4 1 6
C4 5 C1
5 0 6
C5 5 C0
6
C5 5 C2 6 C4 5 C1 6 C5 5 C0
P( E ) 11
C5
6 10 15 5 6 1

462
141

462
Hence, the correct option is (A).

74. With respect to origin B, let BA a and BC c so that c a 0.

c c a c a c
Now, BD and BE BE and AD are perpendicular to each other a 0
2 2 2 2
(c a ) (c 2 a ) 0
| c |2 2 | a |2 c a 0
| c |2 2 | a |2 0 ( c a 0)
2 2
|a| 1 ( AB) 1
2
2

|c | 2 ( BC ) 2
AB 1 1
tan C
BC 2 2
1
C tan 1 .
2
Hence, the correct option is (A).

75. Truth table of ( p q) p is given as follows:

p q pq ( p q) p
T T T T

T F F T

F T F T

F F F T

Hence, ( p q) p is a tautology.
Hence, the correct option is (B)

76. Required mean


1
[( x1 x2 .......... x10 ) (6 12 60)]
10
1 1
[( x1 x2 .......... x10 )] [(6 12 60)]
10 10
6
30 [1 2 10]
10
6 10 11
30
10 2
30 33
63
Hence, the correct option is (C).

77. For the given matrix, we have


0 0 0
2
A2
0 0 0 2
2 0 0
A3
2
0 0
3 0 1 0

3
0 0 1
A2010 ( A3 )670 I 670 I A3
Hence, the correct option is (C).

78.
3 1 3 cos 20 sin 20

sin 20 cos 20 sin 20 cos 40
3 1
2 cos 20 sin 20
2 2

sin 20 cos 20
2[sin 60 cos 20 cos 60 sin 20]

sin 20 cos 20
2sin(60 20)

sin 20 cos 20
2sin 40

sin 20 cos 20
2(2sin 20 cos 20)

sin 20 cos 20
4
Hence, the correct option is (D).

79. We have
2 1 cos x 2 (1 cos x) 1

2
sin x 2
sin x 2 1 cos x
x
2sin 2
2 1

4sin 2 x cos 2 x 2 1 cos x
2 2
1 1

2cos 2
x 2 1 cos x
2
Therefore
2 1 cos x 1 1 1
lim
x 0 2
sin x 2 2 11 4 2
Hence, the correct option is (B).

80. The two curves intersect at (4, 4) which is a vertex of the given square (as shown in figure).
Therefore,
4 4
x2
Required area (shaded portion) 2 x dx (2 1) 1
4
1 2
2 1 3 4
3/ 2 4
2 x x 1
3 1 12 2
4 1
[8 1] (64 8) 1
3 12
28 56 112 56 12 11
1
3 12 12 3
Hence, the correct option is (D).

81. Suppose A = (1, 0, 0,), B = (0, 1, 0) and C(0, 0, 1) are the unit points and the coordinate axes and
consider the cube OADCBEFG (see the figure) so that D = (1, 0, 1), E = (1, 1, 0), F = (1, 1, 1) G = (0, 1,
1). Suppose the given line makes an angle with the diagonal OF. If (l, m, n) are the direction cosines of
the given line, then
(l , m, n) (1, 1, 1) l m n
cos .
(1) 12 12 12 3
Also BD (1, 1, 1), CE (1, 1, 1),and AG (1, 1, 1). Suppose is the angle made by AG with the line,
l m n l mn l mn
we have: cos ;cos and cos =
3 3 3
Therefore,
1
cos2 cos2 cos2 cos2 [(l m n)2 ( l m n)2 (l m n)2 (l m n)2 ]
3
1 4
[4(l 2 m2 n2 )] ( l 2 m2 n 2 1)
3 3
Hence, the correct option is (D).

82. From the hypothesis, we have


a b y / x , c b y / z and b2 ac
Therefore,
b 2 b y / x , b y / z b( y / x ) ( y / z )
y y
2
x z
2 xz y ( x z )
2 xz
y
xz
Therefore, x, y, z are in HP.
Hence, the correct option is (C).

83. A point x = x1 in the domain of f is said to be a stationary point if f ' (x1) = 0.


So, Option (A) is false.

Option (B) is false.

Clearly, from the following figure, at x = x1; f ' (x1) = 0. This implies that y f ( x) has absolute
maximum at x = x1. Also, x = x1 is also a point of local maximum as shown in the figure.

So, Option (C) is true.


If f is continuous on the open interval (a, b); and if f has an absolute extremum on that interval,
then it must occur at a stationary point of f or at a point where f ' (x) does not exist.
For example, let us consider y x ; 1 x 1 at x = 0.
y has local minima and at x = 0, y has absolute minima.
dy
At x = 0, does not exist, that is, x = 0 is one of its critical point.
dx
So, Option (D) is false.
Hence, the correct option is (C).

a 1 1
84. Since the given lines are concurrent, we have : 1 b 1 = 0
1 1 c
Using C2 C1 and C3 C1 (where C1, C2 and C3 are used to denote the three columns), we have
a 1 a 1 a
1 b 1 0 = 0 a(b 1 ) (c 1 ) (1 a ) (c 1) + (1 a ) (1 b) = 0
1 0 c 1
a(1 b) (1 c ) + (1 a) (1 c) + (1 a) (1 b) = 0
a 1 1
Dividing by (1 a) (1 b) (1 c), we get:+ + =0
1 a 1 b 1 c
1 1 1
Adding 1 on both sides, we get: + + =1
1 a 1 b 1 c

Hence, the correct option is (A).

85. P (A / B) = P (A) A and B are independent P (A / B) = P (A) P (B)


P (B/A) = P (B). So option (A) not correct.
P( A ' B) P( B) P( A) P( B)
P( A / B) P( A ') . So option (B) is correct.
P( B) P( B)
P( A ' B ') P( A B) 1 P( A) P( B) P( A B) P( A) P( B)
Also, P( A '/ B ') P( A) . So
P( B ') P( B) P( B) P( B)
option (C) not correct.

And we know that, P (A B) P (A) + P (B). So option (D) not correct.


Hence, the correct option is (B).

86. We know that,


2
t2 t1
t
2
or t1 t2 (1)
t1
Also POQ 90
2at 2at
POQ = 90 11 22 1
at at
2 2
t1t2 4 (2)
From Eq. (1), we have
t12 t1t2 2 t12 4 2
which is written as [from Eq. (2)]
t12 t1t2 2 t12 4 2
t12 2
Hence, the correct option is (A).

87. We have
1 1 3 2
x 3
2, y 2
t t 2t t
Therefore,
dx 3 2

dt t 4 t 3

dy 3 2
and 3 2
dt t t
Therefore,
dy dy dx

dx dt dt
1 3 2
2
tt t
t
1 3 2
2 2
t t t
So
3
dy dy
x xt 3 1 t 1
dx dx
Since
dy
xt 3 1 t and t
dx
Hence, the correct option is (B).

88.
3(2 cos i sin )
x iy
(2 cos ) 2 sin 2
3(2 cos ) i(3sin )

5 4cos

Comparing the real and imaginary portions, we get


3(2 cos ) 3sin
x , y
5 4cos 5 4cos
Squaring and adding values of x and y, we get
9(2 cos ) 2 9sin 2
x2 y 2
(5 4cos ) 2
9(5 4cos ) 9

(5 4cos ) 2
5 4cos

Also
12(2 cos ) 9
4x 3 3
5 4cos 5 4cos
Therefore,
x2 y 2 4x 3
Hence, the correct option is (A).

89. Here, a b 3 3; ab 1 .
1 a 2 b2 c 2 2
cos cos 45 a 2 b2 c 2 [(a b)2 2ab] c 2 2 (27 2) c2
2 2ab 2
2 25 c2 c2 25 2 c 25 2
Therefore, the perimeter of the triangle is :a + b+ c 3 3 25 2
Therefore, k = 2.
Hence, the correct option is (B).

5 3/2
90. Put x5/2 = t .Then: x dx dt Now
2 .
x3/2 2 dt 2 2
dx log t a5 t 2 C log x5/2 a5 x5 C
a5 ( x5/2 )2 5 (a5/2 )2 t 2 5 5
Hence, the correct option is (C).

You might also like